Every sequence of real bounded functions has convergent sub?

Click For Summary
The discussion centers on the convergence of subsequences of real bounded functions when evaluated at a finite set of rational points. The user outlines a method to show that for each rational point in the set, a converging subsequence can be constructed, leading to convergence at each point. There is a suggestion that instead of removing the question, it would be more beneficial to share the findings with the forum community. The conversation highlights the importance of collaborative problem-solving and knowledge sharing in mathematical discussions. Overall, the user has reached a conclusion about the convergence of subsequences but is encouraged to contribute their insights instead of withdrawing the question.
RBG
Messages
14
Reaction score
0
I figured it out... how do I remove this question?
 
Last edited:
Physics news on Phys.org
I don't know the answer, but I have an answer in the case ##x## belongs to finite set of rationals ##S = \{ x_1,...,x_p\} \subset \mathbb{Q}##. It may or may not be useful for what you want to prove.

  1. ## \{f_n(x_1) \}## is a real bounded sequence so there is a converging subsequence ## \{f_{\sigma_1(n)}(x_1) \}## that converges to ##f(x_1)##
  2. ## \{f_{\sigma_1(n)}(x_2) \}## is a real bounded sequence so there is a converging subsequence ## \{f_{(\sigma_2 \circ \sigma_1)(n)}(x_2) \}## that converges to ##f(x_2)##. Furthermore ## \{f_{(\sigma_2 \circ \sigma_1)(n)}(x_1) \}## converges to ##f(x_1)## as a subsequence of ## \{f_{\sigma_1(n)}(x_1) \}##.
  3. Repeating this process, you have ##\{ f_{(\sigma_p\circ ... \circ \sigma_1)(n)}(x_i)\} ## convergerges to ##f(x_i)## for all ##i = 1... p##.
 
RBG said:
I figured it out... how do I remove this question?
Why remove the question?

Wouldn't it be more helpful to other members of the forum to leave the question, and post the answer (or at least an outline of the answer)?
 
Last edited:
Question: A clock's minute hand has length 4 and its hour hand has length 3. What is the distance between the tips at the moment when it is increasing most rapidly?(Putnam Exam Question) Answer: Making assumption that both the hands moves at constant angular velocities, the answer is ## \sqrt{7} .## But don't you think this assumption is somewhat doubtful and wrong?

Similar threads

  • · Replies 7 ·
Replies
7
Views
2K
Replies
15
Views
2K
  • · Replies 13 ·
Replies
13
Views
2K
  • · Replies 2 ·
Replies
2
Views
2K
  • · Replies 5 ·
Replies
5
Views
2K
Replies
4
Views
2K
Replies
6
Views
3K
Replies
8
Views
3K
  • · Replies 8 ·
Replies
8
Views
3K
Replies
3
Views
2K